Difference between revisions of "2002 AMC 8 Problems/Problem 14"

(Solution #1)
m (Solution 2)
(9 intermediate revisions by 5 users not shown)
Line 1: Line 1:
 
+
== Problem ==
== Problem 14 ==
 
  
 
A merchant offers a large group of items at <math>30\%</math> off. Later, the merchant takes <math>20\%</math> off these sale prices and claims that the final price of these items is <math>50\%</math> off the original price. The total discount is
 
A merchant offers a large group of items at <math>30\%</math> off. Later, the merchant takes <math>20\%</math> off these sale prices and claims that the final price of these items is <math>50\%</math> off the original price. The total discount is
Line 7: Line 6:
 
<math> \text{(A)}\ 35\%\qquad\text{(B)}\ 44\%\qquad\text{(C)}\ 50\%\qquad\text{(D)}\ 56\%\qquad\text{(E)}\ 60\% </math>
 
<math> \text{(A)}\ 35\%\qquad\text{(B)}\ 44\%\qquad\text{(C)}\ 50\%\qquad\text{(D)}\ 56\%\qquad\text{(E)}\ 60\% </math>
  
==Solution #1==
+
==Solution==
 
+
===Solution 1===
Let's assume that each item is &#036;100. First we take off <math>30\%</math> off of &#036;100. &#036;100\cdot0.70=<math> &#036;70
 
  
Next, we take off the extra </math>20\%<math> as asked by the problem. &#036;70\cdot0.80=&#036;56
+
Let's assume that each item is <math>100</math> dollars. First we take off <math>30\%</math> off of <math>100</math> dollars. <math>100\cdot0.7=70</math>
  
So the final price of an item is &#036;56. We have to do </math>100-56<math> because </math>56<math> was the final price and we wanted the discount.
+
Next, we take off the extra <math>20\%</math> as asked by the problem. <math>70\cdot0.80=56</math>
  
</math>100-56=44<math> so the final discount was </math>44\%<math>
+
So the final price of an item is \$56. We have to do <math>100-56</math> because <math>56</math> was the final price and we wanted the discount.
  
</math> \text{(A)}\ 35\%\qquad\boxed{\text{(B)}\ 44\%}\qquad\text{(C)}\ 50\%\qquad\text{(D)}\ 56\%\qquad\text{(E)}\ 60\% $
+
<math>100-56=44</math> so the final discount was <math>\boxed{\text{(B)}\ 44\%}</math>.
  
==Solution #2==
+
===Solution 2===
  
Assume the price was &#036;100. We can just do <math>100\cdot0.7\cdot0.8=56</math> and then do <math>100-56=44</math> That is the discount percentage wise.
+
Assume the price was \$100. We can just do <math>100\cdot0.7\cdot0.8=56</math> and then do <math>100-56=\boxed{\text{(B)}\ 44\%}</math>. That is the discount percentage wise.
  
<math> \text{(A)}\ 35\%\qquad\boxed{\text{(B)}\ 44\%}\qquad\text{(C)}\ 50\%\qquad\text{(D)}\ 56\%\qquad\text{(E)}\ 60\% </math>
+
==See Also==
 +
{{AMC8 box|year=2002|num-b=13|num-a=15}}
 +
{{MAA Notice}}

Revision as of 14:59, 25 November 2020

Problem

A merchant offers a large group of items at $30\%$ off. Later, the merchant takes $20\%$ off these sale prices and claims that the final price of these items is $50\%$ off the original price. The total discount is


$\text{(A)}\ 35\%\qquad\text{(B)}\ 44\%\qquad\text{(C)}\ 50\%\qquad\text{(D)}\ 56\%\qquad\text{(E)}\ 60\%$

Solution

Solution 1

Let's assume that each item is $100$ dollars. First we take off $30\%$ off of $100$ dollars. $100\cdot0.7=70$

Next, we take off the extra $20\%$ as asked by the problem. $70\cdot0.80=56$

So the final price of an item is $56. We have to do $100-56$ because $56$ was the final price and we wanted the discount.

$100-56=44$ so the final discount was $\boxed{\text{(B)}\ 44\%}$.

Solution 2

Assume the price was $100. We can just do $100\cdot0.7\cdot0.8=56$ and then do $100-56=\boxed{\text{(B)}\ 44\%}$. That is the discount percentage wise.

See Also

2002 AMC 8 (ProblemsAnswer KeyResources)
Preceded by
Problem 13
Followed by
Problem 15
1 2 3 4 5 6 7 8 9 10 11 12 13 14 15 16 17 18 19 20 21 22 23 24 25
All AJHSME/AMC 8 Problems and Solutions

The problems on this page are copyrighted by the Mathematical Association of America's American Mathematics Competitions. AMC logo.png